آيا شما مايل به راه افتادن تاپيك ماراتن نامساوي مي باشي


  • مجموع رای دهندگان
    339

mahanmath

New Member
ارسال ها
898
لایک ها
701
امتیاز
0
پاسخ : ماراتن نامساوي

خوب ، میخوایم دوباره ماراتن راه بندازیم !
این سوال آخر آقای شریفی سوال سختی هست ، ولی‌ ایدش مثل سوال ۱۹ اینه که اول روی ۲ تا از رادیکال‌ها نابرابری بزنید (مثلا نامساوی Minkowski)







 

math

New Member
ارسال ها
1,129
لایک ها
1,096
امتیاز
0
پاسخ : ماراتن نامساوي

راهنمایی برای سوال 19: اول روی دو تا از کسرها نابرابری به کار ببرید.

[center:2b80b4a96b]


برای اعداد حقیقی و نامنفی
که هیچ دوتایی همزمان صفر نیستند، ثابت کنید

[/center:2b80b4a96b]

این سوال هم با خط بازی نمیشه ؟؟؟

هم متقارنه هم تساوی روی دو سر بازه است ؟؟؟ (مثلا سیگما a رو برابر 1 بگیرید !!!) اگی میشه بگید کل راه حل رو بذارم :53:
 

zz_torna2

New Member
ارسال ها
300
لایک ها
254
امتیاز
0
پاسخ : ماراتن نامساوي

خوب ، میخوایم دوباره ماراتن راه بندازیم !

خیلی وقته که کل ماراتن ها و یا تاپیک کاری از گروه طلا دارن خاک میخورن
اساتید لطفا این ماراتنو ادامه بدید.:53::1:(بعدا سراغ بقیه تاپیک ها هم میریم!:4:)
از اتحاد

حکم معادل:

که داریم:
 

zz_torna2

New Member
ارسال ها
300
لایک ها
254
امتیاز
0
پاسخ : ماراتن نامساوي

سوال بعدی از طرف استاد mahanmath هستش(خودشون امتحان دارند):
a,b,c, نامنفی اند:
 
آخرین ویرایش توسط مدیر

zz_torna2

New Member
ارسال ها
300
لایک ها
254
امتیاز
0
پاسخ : ماراتن نامساوي

کسی نظری نداره؟!
یا لاگرانژ که راحت حل میشه ولی کسی نظری درمورد حل با نامساوی نداره؟؟
 

math

New Member
ارسال ها
1,129
لایک ها
1,096
امتیاز
0
پاسخ : ماراتن نامساوي

اینم سوال بعدی :

اگر داشته باشیم :
ثابت کنید :



یه سوال خوب دیگه هم دیدم گفتم این رو هم بزارم در ادامه این

اگر داشته باشیم :
حداکثر مقدار
را بیابید


 

zz_torna2

New Member
ارسال ها
300
لایک ها
254
امتیاز
0
پاسخ : ماراتن نامساوي

اینم سوال بعدی :

اگر داشته باشیم :
ثابت کنید :


داریم:


کافیه ثابت کنیم:


که بعد به طور مشابه برای پرانتز دوم مینویسیم و با هم جمع میزنیم:

به علت تقارن در شروع عملیات پایین میشه فرض کرد
:

که در نامساوی اخر از چبیشف استفاده شده.

پس حکم ثابت شد.:3:
 
لایک ها ahd

zz_torna2

New Member
ارسال ها
300
لایک ها
254
امتیاز
0
پاسخ : ماراتن نامساوي

یه سوال خوب دیگه هم دیدم گفتم این رو هم بزارم در ادامه این

اگر داشته باشیم :
حداکثر مقدار
را بیابید


ماکسیمم
هستش و اثباتش هم با همگن سازی+شور هستش و حالت تساوی هم
 

zz_torna2

New Member
ارسال ها
300
لایک ها
254
امتیاز
0
پاسخ : ماراتن نامساوي

مطمئنیئ؟
چون اولا باید حداکثر تون عدد باشه(طبق سوال)
دوما اگه الان z از یک بر روی ریشه سوم 9 کوچکتر باشه سمت راست منفی میشه که؟؟!
:3:
 

zz_torna2

New Member
ارسال ها
300
لایک ها
254
امتیاز
0
پاسخ : ماراتن نامساوي

toع.ح.د: البته به طور دقیق تر و کامل تر با همگن سازی +شور max عبارت داده شده میشه:
که شاید منظور شما این هستش.
(بازم مگم باید به عدد باشه پس میشه 1/4 .:3:
 
لایک ها ahd

MBGO

New Member
ارسال ها
247
لایک ها
104
امتیاز
0
پاسخ : ماراتن نامساوي





 

zz_torna2

New Member
ارسال ها
300
لایک ها
254
امتیاز
0
پاسخ : ماراتن نامساوي

این تعمیم نامساوی حسابی-هندسی هستش.از ینسن استفاده کیند و تابع لگاریتم.........:3:
 

MBGO

New Member
ارسال ها
247
لایک ها
104
امتیاز
0
پاسخ : ماراتن نامساوي

منبعی برای اثباتش میدونید؟ + اثباتی غیر از این چیزی که گفتید میدونید داره یا نه؟
ممنون.
 

zz_torna2

New Member
ارسال ها
300
لایک ها
254
امتیاز
0
پاسخ : ماراتن نامساوي

نه ندیدم؟!!:3:

ps:از استاد mahanmath پپرس حتما میدونن
 

MBGO

New Member
ارسال ها
247
لایک ها
104
امتیاز
0
پاسخ : ماراتن نامساوي

اثباتش رو چی با ینسن؟

1:




2:

 
بالا